Cese 2013, 5

Tutti i problemi che presentino una figura (calcolo delle aree e dei perimetri, similitudini, allineamenti, concorrenze, ecc...)
Rispondi
Livex
Messaggi: 994
Iscritto il: 15/03/2013, 15:33

Cese 2013, 5

Messaggio da Livex »

Dato un triangolo isoscele [tex]ABC[/tex] con [tex]AB = AC[/tex] e [tex]\angle BAC < 60°[/tex], sia [tex]D[/tex] il punto su [tex]AC[/tex] tale che [tex]\angle DBC = \angle BAC[/tex], sia [tex]E[/tex] l’intersezione dell’asse di [tex]BD[/tex] con la retta parallela a [tex]BC[/tex] passante per [tex]A[/tex], e sia [tex]F[/tex] il punto sulla retta [tex]AC[/tex], dalla parte di [tex]A[/tex] rispetto a [tex]C[/tex], tale che la lunghezza di [tex]AF[/tex] sia il doppio della lunghezza di [tex]AC[/tex].
Infine, siano [tex]r[/tex] la perpendicolare ad [tex]AB[/tex] condotta da [tex]F[/tex], [tex]s[/tex] la perpendicolare ad [tex]AC[/tex] condotta da [tex]E[/tex], e [tex]t[/tex] la retta [tex]BD[/tex]. Dimostrare che:
(a) le rette [tex]EB[/tex] e [tex]AC[/tex] sono parallele;
(b) le rette [tex]r,s,t[/tex] concorrono.
Testo nascosto:
(a).

Siano [tex]\alpha=\angle BAC= \angle DBC[/tex] e [tex]\beta=\angle BCD=\angle BAE[/tex]. Sia [tex]O[/tex] il centro della circoscritta a [tex]\triangle BDA[/tex], sta quindi sull'asse di [tex]BD[/tex]. Sia [tex]X[/tex] intersezione dell'asse di [tex]BD[/tex] con [tex]AB[/tex].
Chiaramente [tex]\angle BDC= \beta[/tex] per differenza di angoli, ovviamente [tex]\angle ABD=\beta- \alpha[/tex], dunque [tex]\angle AXE=180-(\beta - \alpha +90)=90+ \alpha - \beta[/tex] e infine [tex]\angle OEA=180-(\angle BAE + \angle AXE)=90- \alpha[/tex]

Avendo tutti gli angoli uguali, [tex]\triangle BDC \approx \triangle ABC[/tex] quindi possiamo scrivere [tex]DC \ : \ BC=BC \ : \ AC[/tex] ovvero [tex]BC^2=AC \cdot DC[/tex], cioè, visto che [tex]C[/tex] è un punto esterno, [tex]AC[/tex] è secante mentre [tex]BC[/tex] è tangente alla circoscritta a [tex]\triangle BDA[/tex] (perchè la potenza di C calcolata con la retta [tex]BC[/tex] è [tex]BC^2[/tex]), se è tangente, allora [tex]OB \perp BC[/tex] dunque [tex]\angle OBD=90-\alpha[/tex] ma [tex]\angle OBD= \angle ODB[/tex] quindi [tex]\angle OAD=\angle ODA=180-(\beta + 90- \alpha)=90+ \beta - \alpha[/tex], infine [tex]\angle OAE=\beta + \alpha - \angle OAD=2 \beta -90=90 - \alpha=\angle OEA[/tex], dunque [tex]\triangle OEA[/tex] è isoscele, allora [tex]E[/tex] sta sulla circoscritta e quindi [tex]BDAE[/tex] ciclico.

Infine [tex]\angle BEA=\angle BED+ \angle DEA=\angle BAD + \angle DBA= \alpha + \beta - \alpha= \beta[/tex] perchè [tex]\angle BED, \angle BAD[/tex] e [tex]\angle DBA, \angle DEA[/tex] insistono sulle stesse corde, dunque anche [tex]\triangle BEA[/tex] è isoscele, in particolare è congruente ad [tex]\triangle ABC[/tex] e quindi [tex]BE \| AC[/tex]

b

Fatta la costruzione, sia [tex]Z[/tex] intersezione di [tex]r, AB[/tex], sia [tex]J[/tex] intersezione di [tex]AB, s[/tex], sia[tex]W[/tex] intersezione di [tex]s, r[/tex] e [tex]Y[/tex] intersezione di [tex]t,r[/tex].
La tesi equivale a dimostrare [tex]W \equiv Y[/tex].
Si vede subito che [tex]\angle WJZ=90 -\alpha[/tex] e quindi [tex]\angle EWZ= \alpha[/tex].
Inoltre il quadrilatero[tex]EBYZ[/tex] è ciclico perchè i triangoli [tex]EBY, BYZ[/tex] sono rettangoli e hanno l'ipotenusa in comune. Dunque [tex]\angle EYZ= \angle EBZ= \alpha[/tex] perchè [tex]\triangle EBA \equiv \triangle ABC[/tex].
Cioè [tex]\angle EYZ=\angle EWZ[/tex], ma [tex]W,Y[/tex] stanno sulla stessa retta e nello stesso semipiano rispetto a [tex]EZ[/tex], perciò sono lo stesso punto e la tesi è dimostrata.
Avatar utente
Giovanni98
Messaggi: 1255
Iscritto il: 27/11/2014, 14:30

Re: Cese 2013, 5

Messaggio da Giovanni98 »

a)
Testo nascosto:
Spostiamoci in un sistema di riferimento cartesiano e wlog $A=(\dfrac{1}{2},a) , C=(0,0) , B=(1,0)$. Notiamo che poichè $\angle DBC = \angle BAC$ vale $\angle BDC = \angle DCB$ quindi $BD=BC$. A questo punto , poichè la retta passante per $AC$ ha equazione $y = 2ax$ si ha che $D=(d,2ad)$ ove $d$ è ovviamente un reale positivo. A questo punto imponiamo l'uguaglianza $BD = BC \Rightarrow BD^2 = BC^2 \Rightarrow 1+d^2-2d + 4a^2d^2 = 1$ che ha come soluzioni $d=0$ e $d=\dfrac{2}{1+4a^2}$. Poichè $d=0$ da il punto $C$ bisogna ovviamente prendere l'altra soluzione, quindi $D=(\dfrac{2}{1+4a^2},\dfrac{4a}{1+4a^2})$. Si prenda ora $Q = (\dfrac{3}{2},a)$, in modo quindi che il quadrilatero $AQBC$ sia un parallelogrammo. A questo punto la tesi equivale al dimostrare che $QM \perp DC$, (ove $M$ è il punto medio di $DC$), e che quindi $Q \equiv E$.

Prendiamo la retta passante per $DB$, essa ha come coefficiente angolare $\dfrac{4a}{1-4a^2}$ mentre la retta passante per $QM$ ha come coefficiente angolare $-\dfrac{1-4a^2}{4a}$ quindi uno corrisponde all'antireciproco dell'altro, quindi le due rette sono effettivamente perpendicolari fra loro. (Ti ho risparmiato quei due conti (veramente due) per determinarli perchè sono certo che tu sappia come farli e verificarli).
b)
Testo nascosto:
Siano $R:=r \cap AB$ , $S:= s\cap AC$ , $P:=r \cap s$ e $Q:=s \cap t$.

Lemma 1 : $\angle FEA = 90$.
Dimostrazione : Sia $C'$ il punto medio di $FA$. Poichè $FA=2AC$ vale $FC'=C'A=AC$ e poichè $EACB$ parallelogramma vale $\angle C'AE = \beta \Rightarrow EC' = FC'$ e quindi $FEA$ è triangolo rettangolo con ipotenusa $FA$.

Lemma 2 : $\angle FEP = \beta$.
Dimostrazione : Dal lemma 1 si ha $FEA \approx FSE \Rightarrow \angle FES = \angle FAE = \beta$ , quindi poichè $E,S,P$ allineati vale $FES=FEP = \beta$

Lemma 3 : $PRSA$ è ciclico.
Dimostrazione : Poichè $ES \perp FA$ e $AR \perp FP$ si ha $\angle ASP = \angle ARP = 90$ e quindi $PRSA$ è ciclico. Questo lemma ha come diretta conseguenza $\angle FPE = \alpha$ che comporta $\angle EFP = \beta$.

Lemma 4 : $\angle FDQ = \beta$.
Dimostrazione : Abbiamo dimostrato che $BDC = \beta$ e che $EDB \equiv ACB$ che comporta $ADE = \alpha$ , quindi poichè $FDQ + ADF + EDB = 180$ vale $FDQ = \beta$.

Dal lemma 4 vale $FEDQ$ ciclico, ma quindi se $Q \not \equiv P$ la retta $s$ interseca la circoscritta a $FED$ in $3$ punti, cosa naturalmente impossibile, e poichè $E$ è sicuramente diverso da $P,Q$ deve valere $P \equiv Q$ e quindi $s,t,r$ concorrono.
Livex
Messaggi: 994
Iscritto il: 15/03/2013, 15:33

Re: Cese 2013, 5

Messaggio da Livex »

Il punto a) in analitica è classe :D
Per curiosità, quando ci hai messo a farlo?
Avatar utente
Giovanni98
Messaggi: 1255
Iscritto il: 27/11/2014, 14:30

Re: Cese 2013, 5

Messaggio da Giovanni98 »

Ti giuro 2 minuti.
Saro00
Messaggi: 127
Iscritto il: 26/02/2015, 17:59
Località: Milano Periferia

Re: Cese 2013, 5

Messaggio da Saro00 »

Alternativamente il punto a) si overkilla con il teorema dei seni per dimostrare che due circonferenze con un lato in comune (quali?) hanno stesso raggio...
Un giorno di questi mi metteranno in prigione per aver stuprato troppi problemi.
Rispondi